LSAT and Law School Admissions Forum

Get expert LSAT preparation and law school admissions advice from PowerScore Test Preparation.

 lenihil
  • Posts: 35
  • Joined: Apr 27, 2020
|
#75659
Dear Powerscore,

This is Question #18, PT17S3. Is (D) a viable weakener? I mean something like "Because few readers were buying fantasy fiction, during the last 20 years, the publishers / booksellers of fantasy fiction created an advertising campaign".

Thank you in advance!
 Frank Peter
PowerScore Staff
  • PowerScore Staff
  • Posts: 99
  • Joined: May 14, 2020
|
#75740
HI Leni,

Yes, looks to me like you've correctly identified a weakener answer choice. This is a cause and effect argument - the booksellers are saying that sales have gone up because of increased favorable attention from reviewers. (D) gives us reason to doubt that - now that there is a potential alternate cause in the picture (advertising), we may need additional information to feel certain that it was in fact favorable reviews, and not advertising, that led to the increase in sales.
 lenihil
  • Posts: 35
  • Joined: Apr 27, 2020
|
#75749
Dear Frank,

Now I get it. It weakens the original argument by showing that there is a potential alternate cause.

Thank you very much. :ras: :ras:
 w35t
  • Posts: 6
  • Joined: Jun 22, 2020
|
#77072
Could someone explain why E is the correct answer to this question?

Thank you
 Jeremy Press
PowerScore Staff
  • PowerScore Staff
  • Posts: 1000
  • Joined: Jun 12, 2017
|
#77099
Hi w35t,

We're looking for an answer that strengthens the booksellers' explanation that "increased sales of fantasy books written for adults can be traced to the increased favorable attention given the genre by book reviewers." This is a causal explanation: the booksellers are saying that book reviewers (and their increased favorable attention) caused the rise in sales of fantasy books.

Answer choice E strengthens that cause and effect claim by putting the increased favorable attention of book reviewers at the very beginning of when the rise in sales of fantasy books began (20 years ago). This is what we would expect from a causal relationship: that the cause does in fact occur before the effect. And thus the answer strengthens the causal claim the booksellers are making.

I hope this helps!

Jeremy
 KG!
  • Posts: 69
  • Joined: May 26, 2020
|
#102492
Why would answer choice E not be a premise booster. I chose it due to process of elimination, but felt uncomfortable with the asnswer choice.
 Adam Tyson
PowerScore Staff
  • PowerScore Staff
  • Posts: 5153
  • Joined: Apr 14, 2011
|
#103066
I'm not sure what you mean by a premise booster, KG!, or why that would be a problem. Answer E supports the booksellers' explanation by placing the alleged cause before the alleged effect, and that supports the causal relationship.

Get the most out of your LSAT Prep Plus subscription.

Analyze and track your performance with our Testing and Analytics Package.